Quantcast
Channel: Limiting distribution of $\frac1n \sum_{k=1}^{n}|S_{k-1}|(X_k^2 - 1)$ where $X_k$ are i.i.d standard normal - Cross Validated
Browsing all 2 articles
Browse latest View live

Image may be NSFW.
Clik here to view.

Answer by Sextus Empiricus for Limiting distribution of $\frac1n...

When I simulate the distribution then I get something that resembles a Laplace distribution. Even better seems to be a q-Gausian (the exact parameters you would have to find using theory). I guess...

View Article



Image may be NSFW.
Clik here to view.

Limiting distribution of $\frac1n \sum_{k=1}^{n}|S_{k-1}|(X_k^2 - 1)$ where...

Let $(X_n)$ be a sequence of i.i.d $\mathcal N(0,1)$ random variables. Define $S_0=0$ and $S_n=\sum_{k=1}^n X_k$ for $n\geq 1$. Find the limiting distribution of $$\frac1n...

View Article
Browsing all 2 articles
Browse latest View live




Latest Images